Solving a Matrix Question: -8x-5y-9z=4

  • Thread starter Thread starter Melawrghk
  • Start date Start date
  • Tags Tags
    Matrix
Click For Summary
The discussion revolves around solving a matrix equation, starting with the equation -8x - 5y - 9z = 4. The initial solution involves expressing x in terms of y and z, leading to the conclusion that multiple solutions exist due to the presence of three variables and only one equation. The user then presents a new system of equations and a matrix they have formed, seeking assistance with further reduction. Guidance is provided to reduce the matrix to its reduced-row echelon form and to perform back substitution to find the values of x, y, and z. The conversation emphasizes the importance of systematic approaches in solving linear equations and matrix problems.
Melawrghk
Messages
140
Reaction score
0

Homework Statement



Solve: -8x-5y-9z=4

The Attempt at a Solution



I just need to know if I'm doing this correctly...

There is only one equation given, so I expressed x in terms of y and z and got:
x=-5/8y-9/8z-1/2
then I set y=s and z=t
x=-(5/8)s -(9/8)t -1/2

so I'd enter the answers as:
x = -(5/8)s-(9/8)t-1/2
y = s
z = t

Is that correct? I only have one attempt for the online assignment.

Thanks!
 
Physics news on Phys.org
Try checking it yourself. You have three variables and one equation, so there must be multiple solutions as you suggested. You should be able to pick arbitrary values for "s" (y) and "t" (z) to determine your "x". Do they satisfy the given equation?
 
Yeah it looks ok.
 
Thanks! It was right. I have another problem though. This one I'm actually stuck on...

solve the system:
6A-2B+3C=4
1A+6B-6C=20
3A-1B-2C=9

I made the matrix:
6, -2, 3, 4
1, 6, -6, 20
3, -1, -2, 9

And through some simple things I got to:
1, -13, 11, -31
0, 19, -17, 51
0, 0, 5, -14

After which I don't know what to do... Help?
 
Well, you've reduced the matrix to a row-echelon form. Why not perform further row reduction to reduce it to its reduced-row echelon from? Remember that the leading entry of every pivot column of a RREF matrix is 1.
 
Assuming that is correct (I haven't checked the calculations), those now correspond to x- 13y+ 11z= -31, 19y- 17z= 51, and 5z= -14.

Now you can "back substitute". Solve the last equation for z, substitute that into the second equation and solve for y, then substitute those two values for y and z into the first equation and solve for x.
 
Question: A clock's minute hand has length 4 and its hour hand has length 3. What is the distance between the tips at the moment when it is increasing most rapidly?(Putnam Exam Question) Answer: Making assumption that both the hands moves at constant angular velocities, the answer is ## \sqrt{7} .## But don't you think this assumption is somewhat doubtful and wrong?

Similar threads

Replies
2
Views
2K
  • · Replies 2 ·
Replies
2
Views
947
  • · Replies 10 ·
Replies
10
Views
2K
  • · Replies 2 ·
Replies
2
Views
1K
  • · Replies 3 ·
Replies
3
Views
1K
  • · Replies 11 ·
Replies
11
Views
2K
Replies
1
Views
1K
Replies
2
Views
1K
  • · Replies 1 ·
Replies
1
Views
2K
  • · Replies 7 ·
Replies
7
Views
2K